Showing two groups are *Not* isomorphic

Click For Summary
To demonstrate that \(\mathbb{Z}_{4}\oplus \mathbb{Z}_{4}\) is not isomorphic to \(\mathbb{Z}_{4}\oplus \mathbb{Z}_{2}\oplus \mathbb{Z}_{2}\), the focus is on the elements of order 4 within each group. The group \(\mathbb{Z}_{4}\oplus \mathbb{Z}_{4}\) contains elements like (1,0) and (0,1), which have order 4, while \(\mathbb{Z}_{4}\oplus \mathbb{Z}_{2}\oplus \mathbb{Z}_{2}\) lacks such elements. The order of an element in a direct sum is determined by the least common multiple of the orders of its components. The discussion confirms the calculation of the order of elements like (2,0) in \(\mathbb{Z}_{4}\oplus \mathbb{Z}_{4}\) as correct, reinforcing the understanding of group structure differences. This analysis ultimately supports the conclusion that the two groups are not isomorphic.
DeldotB
Messages
117
Reaction score
8

Homework Statement


Good day,

I need to show:

\mathbb{Z}_{4}\oplus \mathbb{Z}_{4}is not isomorphic to \mathbb{Z}_{4}\oplus \mathbb{Z}_{2}\oplus \mathbb{Z}_{2}

Homework Equations



None

The Attempt at a Solution



I was given the hint that to look at the elements of order 4 in a group. I know \mathbb{Z}_{4}\oplus \mathbb{Z}_{4} will have the elements: (0,0)(0,1)(0,2)(0,3)(1,0)(1,1)...(3,3).

Im a little confused on how to find the order of say (1,2) in \mathbb{Z}_{4}\oplus \mathbb{Z}_{4}.
I know how to find the order of say <3> in \mathbb{Z}_{4} (order=4/gcd(3,4)=4) but how can I do it with the direct sum elements?

Thanks in advance!
 
Physics news on Phys.org
Can anyone tell me this is correct:

In Z4 direct sum Z4, say we look at the element (2,0)
Since <2> has order 2 in Z4 and <0> has order one in Z4, it follows that (2,0) has order 2 because the lcm(2,1) is 2?
 
DeldotB said:
Can anyone tell me this is correct:

In Z4 direct sum Z4, say we look at the element (2,0)
Since <2> has order 2 in Z4 and <0> has order one in Z4, it follows that (2,0) has order 2 because the lcm(2,1) is 2?

Correct. The conclusion is clearly correct since (2,0)+(2,0)=(0,0). Try some others until you can see why that rule works.
 
Last edited:
Question: A clock's minute hand has length 4 and its hour hand has length 3. What is the distance between the tips at the moment when it is increasing most rapidly?(Putnam Exam Question) Answer: Making assumption that both the hands moves at constant angular velocities, the answer is ## \sqrt{7} .## But don't you think this assumption is somewhat doubtful and wrong?

Similar threads

  • · Replies 3 ·
Replies
3
Views
2K
  • · Replies 1 ·
Replies
1
Views
2K
  • · Replies 5 ·
Replies
5
Views
2K
  • · Replies 4 ·
Replies
4
Views
2K
  • · Replies 3 ·
Replies
3
Views
2K
  • · Replies 7 ·
Replies
7
Views
2K
  • · Replies 1 ·
Replies
1
Views
2K
  • · Replies 1 ·
Replies
1
Views
2K
  • · Replies 9 ·
Replies
9
Views
2K
  • · Replies 14 ·
Replies
14
Views
5K